diff --git a/kapI-5.tex b/kapI-5.tex index c3a02bd..e145ac9 100644 --- a/kapI-5.tex +++ b/kapI-5.tex @@ -146,6 +146,7 @@ $\omega_1 = \Omega$ d.h. $\omega = \omega_0$, d.h. $B_1$-Feld zirkuliert mit der \end{itemize} \section{Transformationsverhalten unter Rotationen} +\label{labelTransfRot} \subsection*{klassisch} Vektor $\vec{a}$ wird gedreht mit Matrix $R_z(\varepsilon)$ um den Winkel $\varepsilon$ um die $z$-Achse \begin{align} diff --git a/kapII-1.tex b/kapII-1.tex index 626246e..c6fe739 100644 --- a/kapII-1.tex +++ b/kapII-1.tex @@ -61,23 +61,23 @@ Wie verhält sich $\diffT{t} \dirac{\psi}{\hat{p}}{\psi}$ ? \end{align} \section{Ortsdatsrellung (Analogie zu Spin $\frac{1}{2}$)} -\begin{tabular}{l|c|c} +\begin{tabular}{l||c|c} & Spin & Teilchen \\ \hline\hline Basis & $\sigma_z \ket{z\pm} = \pm 1 \ket{z\pm}$ & $\hat{x} \ket{x} = x \ket{x}$ \\ \hline Orthogonalität & - $\begin{array}[t]{rl}\braket{z+}{z+} &= 1\\ &= \braket{z-}{z-}\\ \braket{z+}{z-} &= \braket{z-}{z+}\\ &= 0\end{array}$ & + $\begin{array}[t]{r@{\,=\,}l}\braket{z+}{z+} & 1\\ &= \braket{z-}{z-}\\ \braket{z+}{z-} & \braket{z-}{z+}\\ & 0\end{array}$ & $\braket{x'}{x} = \delta(x'-x)$ \\ \hline - Zustände in Basis entwickelt & $\begin{array}[t]{rl} \ket{\psi} &= \one \ket{\psi}\\ &= \ket{z+}\braket{z+}{\psi} + \ket{z-}\braket{z-}{\psi} \end{array}$ & $\begin{array}[t]{rl} \ket{\psi} &= \one \ket{\psi}\\ &= \intgr{-\infty}{+\infty}{\ket{x}\underbrace{\braket{x}{\psi}}_{\psi(x)}}{x}\\ &= \intgr{-\infty}{+\infty}{\psi(x) \ket{x}}{x} \end{array}$ + Zustände in Basis entwickelt & $\begin{array}[t]{r@{\,=\,}l} \ket{\psi} & \one \ket{\psi}\\ & \ket{z+}\braket{z+}{\psi} + \ket{z-}\braket{z-}{\psi} \end{array}$ & $\begin{array}[t]{r@{\,=\,}l} \ket{\psi} & \one \ket{\psi}\\ & \intgr{-\infty}{+\infty}{\ket{x}\underbrace{\braket{x}{\psi}}_{\psi(x)}}{x}\\ & \intgr{-\infty}{+\infty}{\psi(x) \ket{x}}{x} \end{array}$ \end{tabular} \paragraph*{Normierung} \begin{align} 1 \stackrel{!}{=} \braket{\psi}{\psi} &= \intgr{-\infty}{+\infty}{\psi^* \bra{x}}{x} \intgr{-\infty}{+\infty}{\psi(x') \ket{x'}}{x'}\\ - \intgru{\intgru{\psi(x)\psi(x')\underbrace{\braket{x}{x'}}_{\delta(x-x')}}{x'}}{x}\\ + &= \intgru{\intgru{\psi(x)\psi(x')\underbrace{\braket{x}{x'}}_{\delta(x-x')}}{x'}}{x}\\ &= \intgru{\psi^*(x)\psi(x)}{x}\\ &= \intgru{(\psi(x))^2}{x} \end{align} -$\rightarrow$ Zulässige Zustände haben eine ``Wellenfunktion'', die quadrat-integrabel ist. +$\rightarrow$ Zulässige Zustände haben eine ``Wellenfunktion'', die quadrat-integrabel ist.align \paragraph*{Erwartungswert einer Ortsmessung} \begin{align} diff --git a/kapIII-2.tex b/kapIII-2.tex index f0aec5c..073bd9e 100644 --- a/kapIII-2.tex +++ b/kapIII-2.tex @@ -1,4 +1,5 @@ \chapter{Rotationssymetrie im Potential in $d=2$} +\label{labelRotSym2D} \section{Lösung der stationären Schrödingergleichung durch ``Separation der Variablen''} \label{rotSymSGL} Mit den Polarkoordinaten diff --git a/kapIII-3.tex b/kapIII-3.tex index 1c10338..254af5f 100644 --- a/kapIII-3.tex +++ b/kapIII-3.tex @@ -237,5 +237,133 @@ mit \begin{equation} m(n_+,n_-) = n_+ - n_- \end{equation} - %TODO... +Für Rotationsinvarianz gilt: +\begin{align} + [L_z, p^2] &= 0\\ + [L_z, r^2] &= 0\\ + \rightarrow [L_z, V(r)] &= 0 +\end{align} +(gilt analog für $L_x$ und $L_y$) +\begin{equation} + \rightarrow \left[ \frac{\hat{p}^2}{2m} + V(r), L^2 \right] = 0 +\end{equation} +also vertauschen $H$, $L^2$ und $L-z$ und es existiert eine gemeinsame Eigenbasis $\set{\ket{E\,l,m}}$ mit +\begin{align} + H \ket{E\,l,m} &= E \ket{E\,l,m}\\ + L^2 \ket{E\,l,m} &= \hbar^2 l(l+1) \ket{E\,l,m}\\ + L_z \ket{E\,l,m} &= \hbar m \ket{E\,l,m} +\end{align} +Sucht man nach dem Spektrum von $H$ so muss man die möglichen $E$-Werte für festes $l$ und $m$ finden. + +\section{Radialgleichung} +\paragraph{Ziel} $H \ket{\phi} = E \ket{\phi}$ vereinfachen mit +\begin{equation} + p^2 = -\frac{\hbar^2}{2\mu} \left[ \diffPs{r}^2 + \frac{2}{r} \diffPs{r} \right] + \frac{\vec{L}^2}{2\mu r^2} +\end{equation} +\paragraph{Beweis} +\begin{align} + L^2 &= \levicivita{\alpha,\beta,\gamma} x_\beta p_x \levicivita{\alpha,\mu,\nu} x_\mu p_\nu \\ + &= \left(\krondelta{\beta,\mu} \krondelta{x,\nu} - \krondelta{\beta,\nu} \krondelta{\gamma,\beta}\right) x_\beta p_\gamma x_\gamma p_\nu \\ + &= x_\beta \underbrace{p_\gamma x_\beta}_{x_\beta p_\gamma - i\hbar \krondelta{\beta,\gamma}} - x_\beta p_\gamma \underbrace{x_\gamma p_\beta}_{p_\beta x_\gamma + i\hbar \krondelta{\gamma,\beta}} \\ + &= x^2 p^2 - i\hbar x p - \left( (x \cdot p)\underbrace{p \cdot x}_{x \cdot p - 3i\hbar} + i\hbar (x \cdot p) \right) \\ + &= x^2 p^2 - (x \cdot p)^2 + i \hbar x p \label{stern00} +\end{align} +mit +\begin{equation} + \dirac{r,\theta,\phi}{x^2 p^2}{\psi} = r^2 \dirac{r,\theta,\phi}{\hat{p}^2}{\psi} +\end{equation} +und +\begin{align} + \dirac{r,\theta,\phi}{\hat{x} p}{\psi} &= \vec{x} (-i\hbar \vec{\nabla}) \psi(r,\theta,\phi) \\ + &= -i\hbar r \diffPs{r} \psi(r,\theta,\phi) +\end{align} +daraus wird (\ref{stern00}) +\begin{align} + \dirac{r,\theta,\phi}{L^2}{\psi} &= r^2 \cdot 2 \mu \braket{r,\theta,\phi \, H_\text{kin}}{\psi} + \hbar^2 (r\diffPs{r})(r\diffPs{r})\psi(r,\theta,\phi) + \hbar^2(r\diffPs{r})\psi(r,\theta,\phi) \\[15pt] + \rightarrow \dirac{r,\theta,\phi}{\underbrace{H_\text{kin} + V(r)}_{H}}{\psi} &\stackrel{!}{=} E \braket{r,\theta,\phi}{\psi} +\end{align} +für Eigenfunktion +\begin{equation} + \rightarrow \left( -\frac{\hbar^2}{2\mu} \left( \diffPs{r}^2 + \frac{2}{r} \diffPs{r} \right) + \frac{L^2}{2\mu r^2} + V(r) \right) \psi +\end{equation} +mit +\begin{equation} + \ket{\psi} = \ket{E\,l,m} +\end{equation} +ist +\begin{align} + \psi(r,\theta,\phi) &= R_E(r) y_{l,m}(\theta,\phi)\\ + \rightarrow \left( -\frac{\hbar^2}{2\mu} \left( \diffPs{r}^2 + \frac{2}{r} \diffPs{r} - \frac{l(l+1)}{r^2} \right) + V(r) \right) R_E(r) &= E R_E(r) +\end{align} +unabhängig von $m$ (nur abhängig von Gesammtdrehimpuls)! +Als Vergleich: Rotation in 2D: +\begin{equation} + \left( -\frac{\hbar^2}{2\mu} \left( \diffPs{\rho}^2 + \frac{1}{\rho} \diffPs{\rho} - \frac{m^2}{\rho^2} \right) + V(\rho) \right) R_E(\rho) = E R_E(\rho) +\end{equation} +Nun ist +\begin{align} + R_E(r) &= \frac{u(r)}{r}\\ + \diffPs{r}R_E(r) &= \frac{\diffPs{r}u(r)}{r} - \frac{u(r)}{r^2}\\ + \diffPs{r}^2R_E(r) &= \frac{\diffPs{r}^2u(r)}{r} - \frac{2\diffPs{r}u(r)}{r^2} + \frac{2u}{r^3} +\end{align} +einsetzen: +\begin{equation} + \left( -\frac{\hbar^2}{2\mu} \diffPs{r}^2 + \underbrace{\frac{\hbar^2}{2\mu} \frac{l(l+1)}{r^2} + V(r)}_{V_\text{eff}(r)} \right) u(r) = E u(r) \text{ für } r \geq 0 +\end{equation} +gleicht formal der eindimensionalen Schrödingergleichung mit effektivem Potential. Aber $r \geq 0$ beachten (mit $u(r = 0) = 0$)! + +\section{Coulomb-Problem und Wasserstoffatom} +Potential: +\begin{equation} + V(r) = -\frac{Z e^2}{r}; ~~ r = a_0 y +\end{equation} +mit +\begin{equation} + a_0 = \frac{\hbar}{\mu e^2} \text{ (Bohr'scher Radius)}; ~~ E = \frac{e^2}{a_0} \varepsilon +\end{equation} +eingesetzt in die stationre Schrödingergleichung +\begin{equation} + \left( \diffPs{y}^2 - \frac{l(l+1)}{y^2} + \frac{2Z}{y} + \varepsilon \right) u(y) = 0 +\end{equation} +\paragraph{Lösungsstrategie 1} Asymptotik bestimmen, abspalten, Potenzreihenansatz (Tailorreihe!) einsetzen, Konvergenz durch Abbruch führt auf quantisierte Energie (steht in jedem Buch). +\paragraph{Lösungsstrategie 2} Abbildung auf harmonischen Oszillator in 2D:\\ +dazu: +\begin{align} + x^2 &= 2 \lambda y\\ + x \text{ d}x &= \lambda \text{ d}y\\ + \rightarrow \diffPfrac{}{y} = \frac{\lambda}{x} \diffPfrac{}{x} +\end{align} +einsetzen: +\begin{equation} + \left( \diffPs{x}^2 - \frac{(2l+1)^2 - \frac{1}{4}}{x^2} + \frac{4Z}{\lambda} + \frac{2\varepsilon}{\lambda^2} x2 \right) \frac{u(y)}{\sqrt{x}} = 0 +\end{equation} +Erinnerung an 2D H.O. +\begin{equation} + \left( \left( \diffPs{\rho}^2 + \frac{1}{\rho} \diffPs{\rho} - \frac{m^2}{\rho^2} \right) -\frac{\mu \omega^2}{\hbar^2} \rho^2 + E \frac{2m}{\hbar^2} \right) R_{n,m}(\rho) = 0 +\end{equation} +mit $q \equiv \sqrt{\frac{u\omega}{\hbar}}\rho$ +\begin{equation} + \left( \diffPs{q}^2 - \frac{m^2 - \frac{1}{4}}{q^2} - q^2 + 2N + 2 \right) \sqrt{\rho} R_{n,m}(\rho) = 0 +\end{equation} +Korrespondenz: +\begin{center} + \begin{tabular}{c|c} + Coulomb & H.O. \\ \hline + $(2l+1)^2$ & $m^2$ \\ + $\frac{4Z}{\lambda}$ & $2N+2$ \\ + $\frac{2\varepsilon}{\lambda}$ & $-1$ + \end{tabular} +\end{center} +Also sind: +\begin{equation} + \abs{m} = 2l+1 \text{ und } \lambda = \frac{2Z}{N+1} = \frac{2Z}{\abs{m} + 2n_r + 1} +\end{equation} +Energieeigenwerte: +\begin{equation} + \varepsilon = -\frac{\lambda^2}{2} = -2Z^2 \frac{1}{(\abs{m} + 2n_r + 1)^2} = -z^2 \frac{1}{(l + n_r + 1)^2} +\end{equation} +damit das Spektrum: +%\begin{figure}[H] \centering +%\includegraphics{pdf/III/03-05-00.pdf} +%\end{figure} diff --git a/kapIII-4.tex b/kapIII-4.tex index 6ae0761..4264a1a 100644 --- a/kapIII-4.tex +++ b/kapIII-4.tex @@ -8,7 +8,7 @@ mit \begin{equation} D(\phi,\vec{n}) = 1 - i\frac{\phi}{\hbar} J_{\vec{n}} + O(\phi^2) \end{equation} -In () hatten wir die Relation +In (\ref{labelTransfRot}) hatten wir die Relation \begin{equation} [J_x,J_y] = i\hbar J_z \end{equation} @@ -62,3 +62,134 @@ $\beta$-Spektrum ist eingeschränkt wegen: \begin{equation} 0 \leq \dirac{\alpha,\beta}{J_x^2 + J_y^2}{\alpha,\beta} = \dirac{\alpha,\beta}{J^2-J_z^2}{\alpha,\beta} = (\alpha-\beta)^2 \underbrace{\braket{\alpha,\beta}{\alpha,\beta}}_{\geq 0} \end{equation} +Also ist $\alpha \geq \beta^2$ und $\exists \beta_\text{max}$ mit +\begin{align} + J_+ \ket{\alpha,\beta_\text{max}} &= \ket{\zero} & \left| J_- \right.\\ + J_- J_+ \ket{\alpha,\beta_\text{max}} &= 0\\ + \left( J^2 - J_z^2 - \hbar J_z \right) \ket{\alpha,\beta_\text{max}} &= 0\\ + \alpha - \beta_\text{max}^2 \hbar^2 -\beta_\text{max} \hbar &\stackrel{!}{=} 0\\ + \rightarrow \alpha = \beta_\text{max} (\beta_\text{max} + \hbar) +\end{align} +entsprechend: +\begin{align} + J_- \ket{\alpha,\beta_\text{min}} &= \ket{\zero}\\ + ... \rightarrow \alpha &= \beta_\text{min} (\beta_\text{min} - \hbar) +\end{align} +Daraus folgt: +\begin{equation} + \beta_\text{max}^2 + \beta_\text{max} \hbar = \beta_\text{min}^2 + \beta_\text{min} \hbar +\end{equation} +quadratische Gleichung mit den 2 Lösungen: +\begin{align} + \beta_\text{max} &= \beta_\text{min} - \hbar &\text{(irrelevant)}\\ + \beta_\text{max} &= -\beta_\text{min} +\end{align} +\begin{align} + \left( J_+ \right)^k \ket{\alpha,\beta_\text{mix}} &= \const \ket{\alpha,\beta_\text{max}}\\ + \rightarrow \beta_\text{max} &= \beta_\text{min} + \hbar \cdot k +\end{align} +daraus ergibt sich mit $k = 0,1,2,...$: +\begin{equation} + \boxed{\beta_\text{max} = \frac{\hbar}{2} \cdot k} +\end{equation} +und +\begin{equation} + \alpha = \beta_\text{max}^2 + \beta_\text{max} \hbar = \hbar^2 \left( \frac{k}{2} + 1 \right) \frac{k}{2} +\end{equation} +Zusammenfassung: +\begin{center} + \begin{tabular}{c|c|c|c|c} + $\frac{k}{2}$ & $\beta_\text{max}$ & $\alpha$ & $\ket{\alpha,\beta_\text{max}}$ & Anzahl $\ket{\alpha,\beta}$ \\ \hline + $0$ & $0$ & $0$ & $\ket{0,0}$ & $1$ \\ \hline + $\frac{1}{2}$ & $\frac{1}{2}\hbar$ & $\frac{3}{4}\hbar^2$ & $\ket{\frac{3}{4},\frac{1}{2}}$ & $2$ \\ \hline + $1$ & $\hbar$ & $2\hbar^2$ & $\ket{2,1}$ & $3$ \\ \hline + $\frac{3}{2}$ & $\frac{3}{2}\hbar$ & $\frac{15}{4}\hbar^2$ & $\ket{\frac{15}{4},\frac{3}{2}}$ & $4$ + \end{tabular} +\end{center} +Wir finden halbzahlige Eigenwerte für $J_z$. Aber: in (\ref{labelRotSym2D}) hatten wir gesehen: +\begin{equation} + J_3 = XP_Y - YP_X +\end{equation} +hat Eigenwerte $m \hbar= (0,\pm 1, \pm 1, ...) \hbar$.\\ +Mit der Notation $j \equiv \frac{k}{2}$ +\begin{align} + \alpha &= \hbar^2 j (j + 1)\\ + \beta &= \hbar m +\end{align} +und +\begin{align} + J^2 \ket{j,m} &= \hbar^2 j (j+1) \ket{j,m} & \left(j = \frac{0,1,2,3,...}{2}\right)\\ + J_z \ket{j,m} &= \hbar m \ket{j,m} & \left(m = -j, -j+1, ..., j\right) +\end{align} +Matrixelemente: +\begin{align} + J_\pm \ket{j,m} &= c_+(j,m) \ket{j,m \pm 1}\\ + \bra{j,m} J_\mp &= c_+^*(j,m) \bra{j,m \pm 1}\\[15pt] + \dirac{j,m}{J_- J_+}{j,m} &= \abs{c_+(j,m)}^2 \underbrace{\braket{j,m+1}{j,m+1}}_1\\ + \dirac{j,m}{J^2 - J_z^2 - \hbar J_z}{j,m} &= \abs{c_+(j,m)}^2\\ + \rightarrow \hbar^2 \left( j (j+1) - m^2 - m \right) &= \abs{c_+(j,m)}^2\\ + \rightarrow c_+ &= \hbar \sqrt{j (j+1) - m^2 - m}\\ + &= \sqrt{(j + m + 1)(j - m)} +\end{align} +genauso +\begin{equation} + c_- = \hbar \sqrt{(j - m + 1)(j + m)} +\end{equation} +Matrixelemente von $J_x$ und $J_y$: +\begin{align} + \dirac{j',m'}{J_x}{j,m} &= \braket{j',m'}{\frac{J_+ + J_-}{2} - j,m}\\ + &= \krondelta{j',j} \left\lbrace \krondelta{m',m+1} c_+(j,m) + \krondelta{m'-1,m} c_-(j,m) \right\rbrace +\end{align} +\definecolor{lgray}{gray}{0.9} +\newcommand{\graycell}{\cellcolor{lgray}} +\begin{itemize} + \item für $J^2 / \hbar^2$ + \begin{center} + \begin{tabular}{c||c|c|c|c|c|c} + $j',m' \backslash j,m$ & $(0,0)$ & $(\frac{1}{2},\frac{1}{2})$ & $(\frac{1}{2},-\frac{1}{2})$ & $(1,1)$ & $(1,0)$ & $(1,-1)$ \\ \hline\hline + $(0,0)$ & $0$ \graycell & $0$ & $0$ & $0$ & $0$ & $0$ \\ \hline + $(\frac{1}{2},\frac{1}{2})$ & $0$ & $\frac{3}{4}$ \graycell & $0$ \graycell & $0$ & $0$ & $0$ \\ \hline + $(\frac{1}{2},-\frac{1}{2})$ & $0$ & $0$ \graycell & $\frac{3}{4}$ \graycell & $0$ & $0$ & $0$ \\ \hline + $(1,1)$ & $0$ & $0$ & $0$ & $2$ \graycell & $0$ \graycell & $0$ \graycell \\ \hline + $(1,0)$ & $0$ & $0$ & $0$ & $0$ \graycell & $2$ \graycell & $0$ \graycell \\ \hline + $(1,-1)$ & $0$ & $0$ & $0$ & $0$ \graycell & $0$ \graycell & $2$ \graycell \\ + \end{tabular} + \end{center} + \item für $J_z / \hbar$ + \begin{center} + \begin{tabular}{c||c|c|c|c|c|c} + $j',m' \backslash j,m$ & $(0,0)$ & $(\frac{1}{2},\frac{1}{2})$ & $(\frac{1}{2},-\frac{1}{2})$ & $(1,1)$ & $(1,0)$ & $(1,-1)$ \\ \hline\hline + $(0,0)$ & $0$ \graycell & $0$ & $0$ & $0$ & $0$ & $0$ \\ \hline + $(\frac{1}{2},\frac{1}{2})$ & $0$ & $\frac{1}{2}$ \graycell & $0$ \graycell & $0$ & $0$ & $0$ \\ \hline + $(\frac{1}{2},-\frac{1}{2})$ & $0$ & $0$ \graycell & $-\frac{1}{2}$ \graycell & $0$ & $0$ & $0$ \\ \hline + $(1,1)$ & $0$ & $0$ & $0$ & $1$ \graycell & $0$ \graycell & $0$ \graycell \\ \hline + $(1,0)$ & $0$ & $0$ & $0$ & $0$ \graycell & $0$ \graycell & $0$ \graycell \\ \hline + $(1,-1)$ & $0$ & $0$ & $0$ & $0$ \graycell & $0$ \graycell & $1$ \graycell \\ + \end{tabular} + \end{center} + \item für $J_x / \hbar$ + \begin{center} + \begin{tabular}{c||c|c|c|c|c|c} + $j',m' \backslash j,m$ & $(0,0)$ & $(\frac{1}{2},\frac{1}{2})$ & $(\frac{1}{2},-\frac{1}{2})$ & $(1,1)$ & $(1,0)$ & $(1,-1)$ \\ \hline\hline + $(0,0)$ & $0$ \graycell & $0$ & $0$ & $0$ & $0$ & $0$ \\ \hline + $(\frac{1}{2},\frac{1}{2})$ & $0$ & $0$ \graycell & $\frac{1}{2}$ \graycell & $0$ & $0$ & $0$ \\ \hline + $(\frac{1}{2},-\frac{1}{2})$ & $0$ & $\frac{1}{2}$ \graycell & $0$ \graycell & $0$ & $0$ & $0$ \\ \hline + $(1,1)$ & $0$ & $0$ & $0$ & $1$ \graycell & $0$ \graycell & $0$ \graycell \\ \hline + $(1,0)$ & $0$ & $0$ & $0$ & $0$ \graycell & $0$ \graycell & $0$ \graycell \\ \hline + $(1,-1)$ & $0$ & $0$ & $0$ & $0$ \graycell & $0$ \graycell & $1$ \graycell \\ + \end{tabular} + \end{center} + \item für $J_y / \hbar$ + \begin{center} + \begin{tabular}{c||c|c|c|c|c|c} + $j',m' \backslash j,m$ & $(0,0)$ & $(\frac{1}{2},\frac{1}{2})$ & $(\frac{1}{2},-\frac{1}{2})$ & $(1,1)$ & $(1,0)$ & $(1,-1)$ \\ \hline\hline + $(0,0)$ & $0$ \graycell & $0$ & $0$ & $0$ & $0$ & $0$ \\ \hline + $(\frac{1}{2},\frac{1}{2})$ & $0$ & $0$ \graycell & $-\frac{i}{2}$ \graycell & $0$ & $0$ & $0$ \\ \hline + $(\frac{1}{2},-\frac{1}{2})$ & $0$ & $\frac{i}{2}$ \graycell & $0$ \graycell & $0$ & $0$ & $0$ \\ \hline + $(1,1)$ & $0$ & $0$ & $0$ & $1$ \graycell & $0$ \graycell & $0$ \graycell \\ \hline + $(1,0)$ & $0$ & $0$ & $0$ & $0$ \graycell & $0$ \graycell & $0$ \graycell \\ \hline + $(1,-1)$ & $0$ & $0$ & $0$ & $0$ \graycell & $0$ \graycell & $1$ \graycell \\ + \end{tabular} + \end{center} +\end{itemize} +Erkennbar ist hier jeweils eine Blockstruktur! diff --git a/math.tex b/math.tex index 8d17421..3ed26ef 100644 --- a/math.tex +++ b/math.tex @@ -37,8 +37,8 @@ \newcommand{\diffTfrac}[2]{\frac{\text{d} #1}{\text{d} #2}} \newcommand{\diffTm}[3]{\diffTfrac{^{#1} #2}{#3^{#1}}} \newcommand{\diffPfrac}[2]{\frac{\partial #1}{\partial #2}} -\newcommand{\intgr}[4]{\int_{#1}^{#2} #3 ~\text{ d}#4} -\newcommand{\intgru}[2]{\int #1 ~\text{ d}#2} +\newcommand{\intgr}[4]{\int_{#1}^{#2} #3 \,\text{d}#4} +\newcommand{\intgru}[2]{\int #1 \,\text{d}#2} \newcommand{\intgrinf}[2]{\intgr{-\infty}{+\infty}{#1}{#2}} \newcommand{\sbk}[1]{\left( #1 \right)} diff --git a/theo2.kilepr b/theo2.kilepr index ad1f38d..7e2b8f3 100644 --- a/theo2.kilepr +++ b/theo2.kilepr @@ -3,7 +3,7 @@ img_extIsRegExp=false img_extensions=.eps .jpg .jpeg .png .pdf .ps .fig .gif kileprversion=2 kileversion=2.0 -lastDocument=kapIII-4.tex +lastDocument=kapIV-1.tex masterDocument= name=Theo2 pkg_extIsRegExp=false @@ -65,7 +65,7 @@ archive=true column=7 encoding=UTF-8 highlight=LaTeX -line=82 +line=148 open=false order=6 @@ -84,15 +84,15 @@ column=12 encoding=UTF-8 highlight=LaTeX line=122 -open=true +open=false order=1 [item:kapII-1.tex] archive=true -column=6 -encoding= +column=113 +encoding=UTF-8 highlight=LaTeX -line=110 +line=67 open=false order=-1 @@ -120,7 +120,7 @@ column=13 encoding=UTF-8 highlight=LaTeX line=39 -open=true +open=false order=2 [item:kapII-5.tex] @@ -129,7 +129,7 @@ column=29 encoding=UTF-8 highlight=LaTeX line=92 -open=true +open=false order=3 [item:kapIII-0.tex] @@ -138,7 +138,7 @@ column=8 encoding=UTF-8 highlight=LaTeX line=0 -open=true +open=false order=4 [item:kapIII-1.tex] @@ -147,42 +147,51 @@ column=15 encoding=UTF-8 highlight=LaTeX line=39 -open=true +open=false order=5 [item:kapIII-2.tex] archive=true -column=49 +column=20 encoding=UTF-8 highlight=LaTeX -line=0 -open=true +line=1 +open=false order=6 [item:kapIII-3.tex] archive=true -column=50 +column=108 encoding=UTF-8 highlight=LaTeX -line=0 -open=true +line=338 +open=false order=7 [item:kapIII-4.tex] archive=true -column=69 +column=0 encoding=UTF-8 highlight=LaTeX -line=8 -open=true +line=195 +open=false order=8 +[item:kapIV-1.tex] +archive=true +column=104 +encoding=UTF-8 +highlight=LaTeX +line=53 +open=true +order=1 + [item:math.tex] archive=true -column=43 +column=34 encoding=UTF-8 highlight=LaTeX -line=37 +line=40 open=false order=2 @@ -206,10 +215,10 @@ order=-1 [item:theo2.tex] archive=true -column=17 +column=24 encoding=UTF-8 highlight=LaTeX -line=47 +line=70 open=true order=0 diff --git a/theo2.tex b/theo2.tex index f249f4d..06d7fb4 100644 --- a/theo2.tex +++ b/theo2.tex @@ -9,7 +9,7 @@ \usepackage{float} \usepackage{pdflscape} \usepackage[pdfborder={0 0 0}]{hyperref} % muss immer als letztes eingebunden werden - +\usepackage{colortbl} \include{math} \include{physics} @@ -48,23 +48,27 @@ \include{kapIII-3} \include{kapIII-4} +\part{Näherungsmethoden} +\label{IV} +\include{kapIV-1} +\include{kapIV-2} + % \part{Übungsmitschrieb} % \label{UE} -\include{ueb1} -\include{ueb2} -\include{ueb3} -\include{ueb4} -\include{ueb5} -\include{ueb6} -\include{ueb7} -\include{ueb8} -\include{ueb9} -\include{ueb10} -\include{ueb11} +% \include{ueb1} +% \include{ueb2} +% \include{ueb3} +% \include{ueb4} +% \include{ueb5} +% \include{ueb6} +% \include{ueb7} +% \include{ueb8} +% \include{ueb9} +% \include{ueb10} +% \include{ueb11} \part{Formelsammlung} \label{FS} \include{formelsammlung} - \end{document}